¿Enredado o desenredado?

Me quedé un poco desconcertado al pensar en dos fermiones entrelazados.

Digamos que tenemos un espacio de Hilbert en el que tenemos dos orbitales fermiónicos a y b . Entonces el espacio de Hilbert H la dimensión de es justo 4 , ya que está atravesado por

{ | 0 , C a | 0 , C b | 0 , C a C b | 0 } ,
dónde C i son los operadores fermiónicos que crean un fermión en orbital i .

Digamos que tenemos un estado C a C b | 0 . Entonces, si divido mi espacio de Hilbert en dos mirando el producto tensorial de los espacios de Hilbert de cada orbital, es decir H = H a H b , entonces mi estado se puede escribir como C a | 0 a C b | 0 b , de lo que es obvio que este estado está desenredado ( | 0 = | 0 a | 0 b ).

Ahora estaba pensando en escribir el estado en primer cuantizado, es decir, una función de onda. Dejar ϕ a ( r ) , ϕ b ( r ) ser las funciones de onda de los orbitales a y b . Después

ψ ( X 1 , X 2 ) = X 1 X 2 | C a C b | 0 = ϕ a ( X 1 ) ϕ b ( X 2 ) ϕ a ( X 1 ) ϕ b ( X 2 ) .
Aquí es donde me confundí. que objeto es ψ ( X 1 , X 2 ) , es decir, ¿a qué espacio de Hilbert pertenece? ¿Qué estamos haciendo exactamente cuando hacemos X 1 X 2 | C a C b | 0 ? ¿Parece que estamos cambiando/expandiendo nuestro espacio de Hilbert al tomar la representación de posición?

Escrito de esta manera, y asumiendo la misma partición H a H b , la naturaleza desenredada del estado original ya no se manifiesta. No estoy seguro de cuál es la partición H a H b incluso significa en este contexto. ¿Sería eso decir ψ ( X 1 , X 2 ) = ψ a ( X 1 , X 2 ) × ψ b ( X 1 , X 2 ) dónde ψ i ( X 1 , X 2 ) es una combinación lineal de ϕ i ( X 1 ) , ϕ i ( X 2 ) ? Esto no me parece correcto.

Independientemente, ahora tengo un estado escrito de dos maneras diferentes pero supuestamente equivalentes, con la misma partición del espacio de Hilbert, pero está desenredado en un sentido y enredado en el otro.

¿Ayuda?

No puede dividir un sector de dos fermiones del espacio de Fock como producto tensorial: los estados subyacentes siempre están antisimétricos
@ Slaviks: lo siento, no entendí muy bien lo que querías decir. Estas diciendo H = H a H b ¿Está Mal? pero así es como construyes estados de Fock multi-fermión. cada operador C i actúa en un espacio de Hilbert que es solo el vacío y una excitación, y así para norte fermiones H = i = 1 norte H i ( 2 norte estados posibles). Aquí solo estoy considerando norte = 2 .
@Slaviks: si aplica una transformación de Jordan-Wigner, obtiene inmediatamente una descomposición del producto tensorial en espacios de Hilbert bidimensionales. Creo que esta pregunta es un debate en curso en la literatura. Existen diferentes enfoques, ya que no parece particularmente fácil tratar con la (anti-)simetrización. En mi opinión, la respuesta debe encontrarse considerando el resultado de las mediciones y sus correlaciones y, por lo tanto, (probablemente) las propiedades de factorización de las matrices de densidad de uno y dos cuerpos en los casos más simples. Véase, por ejemplo, arXiv:0902.1684 y PRA 67, 024301 (2003) y sus refs.
@nervxxx Veo lo que quieres decir. Estoy diciendo que la descomposición del número de ocupación H 1 H 2 no es lo mismo que (lo imposible) la descomposición en el producto tensorial de espacios de Hilbert de una sola partícula. Pero necesito pensar más, especialmente a la luz del comentario de S. Gammelmark.
De hecho, hay mucha literatura en torno a discutir el hecho de que la antisimetrización que se obtiene automáticamente en estados de múltiples fermiones no debe considerarse como un "entrelazamiento" operativamente útil, por ejemplo , Zanardi et al afirman que no es útil discutir el entrecruzamiento sin especificar la manera en que uno puede manipular y sondear sus grados de libertad físicos constituyentes. En este sentido, el entrelazamiento siempre es relativo a un conjunto particular de capacidades experimentales.
@ Slaviks: todavía confundido. Mi H i es el espacio de Hilbert de orbitales individuales, no un espacio de Hilbert de una sola partícula. El número de partículas es indeterminado (pero está restringido a ser 0 2 en este caso), hasta que doy un estado. es decir, no estoy mirando el espacio de Fock de 2 partículas.
Para ampliar el comentario de twistor59: en este caso, una restricción importante es que los fermiones son indistinguibles. En un estado de Bell tradicional, podemos realizar cualquier medición en cualquiera de los dos sistemas de forma individual. Pero en una primera imagen cuantificada de 2 fermiones que ocupan 2 modos, no hay forma de realizar una medición en solo 1 de los fermiones. Es una virtud de la segunda imagen cuantizada que esta aparente restricción se muestra como una confusión: no se puede distinguir entre los dos fermiones porque no son dos objetos diferentes. Sólo cuentan la excitación de los objetos más fundamentales, los modos.
@twistor59 +1 para la referencia de Zanardi, aborda el problema muy bien en la columna 2 de la página 1.

Respuestas (3)

Permítanme recordarles que el espacio de Fock de múltiples fermiones se define como el subespacio antisimétrico (fermiónico) del espacio de Fock completo

Γ a = norte = 0 H norte ,

dónde representa el producto tensorial antisimétrico

v 1 v norte = 1 norte ! pags PAGS norte σ pags v pags ( 1 ) v pags ( norte ) .

Aquí σ pags es el signo de la permutación pags en el grupo de permutaciones PAGS norte .

Por lo tanto, la confusión aquí proviene del hecho de que C a C b | 0 | a b como pareces decir.

Recuerde que los operadores de creación y aniquilación se definen dentro de la representación del número de ocupación, es decir C a C b | 0 = | 11 , donde el primer número denota el número de fermiones en estado a mientras que el segundo denota el número de fermiones en estado b . Por otro lado, los estados escritos en la representación del número de ocupación se definen como estados de base de muchos cuerpos correctamente antisimetrizados (para fermiones), como nos impone la indistinguibilidad de las partículas. Por lo tanto, se definen dentro del espacio fermiónico de Fock. Cualquier libro de texto lo mostrará, eche un vistazo al primer capítulo de Teoría cuántica de muchos cuerpos en física de la materia condensada: una introducción de Bruus y Flensberg, por ejemplo. Para dos fermiones descritos a través de una base de partícula única { | a , | b } una opción posible es:

| 11 = 1 2 ( | a b | b a ) .
Por lo tanto

C a C b | 0 = 1 2 ( C a | 0 1 C b | 0 2 C b | 0 1 C a | 0 2 )

La anticonmutatividad familiar de estos operadores es ahora obvia a partir de esto de

C b C a | 0 = 1 2 ( C b | 0 1 C a | 0 2 C a | 0 1 C b | 0 2 ) = C a C b | 0

De hecho, una de las grandes ventajas de los operadores de creación y aniquilación es que incluyen implícitamente la antisimetría (para los fermiones) de la función de onda.

Punteando esto con X 1 X 2 | obtenemos:

X 1 X 2 | C a C b | 0 = 1 2 ( ϕ a ( X 1 ) ϕ b ( X 2 ) ϕ b ( X 1 ) ϕ a ( X 2 ) ) .

Por lo tanto, no hay inconsistencia, ambas representaciones muestran que las partículas están entrelazadas.

Por otro lado punteando X 1 X 2 | con C a | 0 1 C b | 0 2 simplemente produciría

ψ ( X 1 , X 2 ) = ϕ a ( X 1 ) ϕ b ( X 2 )

Por lo tanto, no hay inconsistencia aquí también, sin embargo, como dije, lo importante para recordar es que

C a C b | 0 C a | 0 1 C b | 0 2

No, eso no es correcto. Con la elección de la partición del espacio de Hilbert que he especificado en el problema, los dos subsistemas están inequívocamente desenredados . Cito, por ejemplo, el uso del corte orbital en el espectro de entrelazamiento de los estados QH (ve a buscarlo) - allí, los estados IQH están desenredados porque tienen la forma | ψ = i C i | 0 .
Próximo, C a C b | 0 = 1 2 ( C a | 0 1 C b | 0 2 C b | 0 1 C a | 0 2 ) Está Mal. El vacío de estado de Fock multipartícula se construye a partir del producto tensorial de vacíos individuales i | 0 i . Entonces surge una representación de estado de Fock al actuar los operadores de creación sobre esto: C a C b | 0 = | 0 i C a | 0 a C b | 0 b | 0 j
La anticonmutatividad surge debido a las relaciones canónicas de anticonmutación { C i , C j } = d i j . Esto entonces resulta en C a C b | 0 = C b C a | 0 . el punto central de la representación del número de estado de Fock es que trata a las partículas inmediatamente como indistinguibles y no tiene una etiqueta no física (las posiciones, X 1 , X 2 , ) para cada partícula como lo haría una primera notación cuantificada (función de onda). La primera notación cuantizada es realmente engorrosa e incómoda, porque lo que hace
es decir que cada partícula es distinguible con etiquetas de posición X 1 , X 2 , , pero luego hace cumplir la regla de que en realidad son indistinguibles y deben ser impares bajo un intercambio de etiquetas, lo que lleva al esquema de determinante/antisimetrización de Slater. Entonces, -1.
@nervxxx, he editado mi respuesta para explicar mejor mi punto. Sin embargo, también me gustaría que me proporcionaras referencias concretas, ya que "ir a buscarlo" no es una.
@nervxxx, en lo que respecta al espacio de Fock multipartícula, expliqué en mi respuesta que para los fermiones se usa el producto tensorial antisimétrico. Por lo tanto para dos fermiones C a C b | 0 = 1 2 ( C a | 0 1 C b | 0 2 C b | 0 1 C a | 0 2 ) es correcto.
Estoy de acuerdo con el resto de sus comentarios y señalo que toda mi respuesta se basa en eso.
es posible que necesite algo de experiencia en física QH, pero el v = 1 El estado IQH es un estado de producto puro . ρ es un estado producto en la base orbital y, por lo tanto, ρ A es un estado puro para cualquier bi-partición orbital (donde todos norte A orbitales en subsistema A están llenos) de este sistema. En consecuencia, la ES (y, por lo tanto, la entropía de entrelazamiento) es trivial y consta exactamente de un nivel.
la esencia es que para un estado de producto puro, con el tipo de partición que describí en mi publicación, no hay enredo, es decir, solo tiene un valor de Schmidt.
Estimado @nervxxx, gracias por los recursos. Entiendo y estoy de acuerdo con estos argumentos. El único punto que estaba tratando de hacer es que si quieres un H = H a H b partición que no puedes usar C a C b | 0 vector, ya que implica una partición antisimétrica. Esto, creo, es la fuente de confusión.
esa declaración es completamente infundada. ¿Por qué no puedes usar el vector? Eliges un vector simplemente del espacio de Hilbert completo H . La partición que impone en el espacio de Hilbert es una elección completamente independiente. Para los recursos que te di, mira: en el nivel más bajo de Landau, el espacio de Hilbert está dado por el producto tensorial de los espacios orbitales de Hilbert H = i = 1 yo H i . El corte orbital viene dado por la elección del particionado H = H a H b ( i = 1 metro H i ) ( i = metro + 1 yo H i ) . El estado IQH, que viene dado por
( i = 1 yo C i ) | 0 , bajo esta partición, se convierte en i = 1 metro C i | 0 i = metro + 1 yo C i | 0 . Así que no hay enredo. Esto es lo que los autores quieren decir con que el corte orbital tiene solo un nivel de Schmidt. En cambio, creo que está bastante confundido con su comprensión del espacio de Fock antisimetrizado.
Le agradecería si pudiera especificar lo que cree que está mal en mi respuesta. ¿Quiso decir eso? C a C b | 0 no implica producto tensorial antisimétrico?
Tenga en cuenta que, por la forma en que planteó la pregunta, tiene muy poco que ver con IQH. Tampoco entra en la naturaleza del enredo de partículas indistinguibles, que se analiza, por ejemplo, en physik.rwth-aachen.de/fileadmin/user_upload/www_physik/… y sqig.math.ist.utl.pt/pub/PaunkovicN/04 -P-tesis.pdf
También me parece que la pregunta que hiciste no es la que querías hacer, y la respuesta a esta última podría estar aquí physics.stackexchange.com/questions/35185/…
No confiaría en la tesis de licenciatura que me diste, tiene errores. la tesis doctoral está bien. mi pregunta tiene mucho que ver con el IQH: solo estoy restringiendo el número de orbitales a 2 con fines ilustrativos, mientras que en el IQHE, en un colector compacto, el número de orbitales es norte o r b . Ok, lo que está mal con tu comentario se especifica en mi segundo comentario en la cadena. Su última afirmación, que C a C b | 0 no es correcto. ¡Estás confundiendo completamente la segunda notación cuantizada y la primera!
Pero de todos modos ahora veo el problema, lamento haber tardado tanto. Mi H no es lo mismo que tu H . Su H es el espacio de Hilbert de una sola partícula. Mi H i es el espacio de Hilbert de la i -ésimo modo fermiónico. Ilustraré esta diferencia contando argumentos. decir que tenemos METRO orbitales (con diferentes números cuánticos). Cada orbital tiene sus operadores de subida y bajada asociados. C i , C i . Entonces, el espacio de Hilbert del orbital (que he estado usando) es de dimensión 2. El espacio de Hilbert completo es el producto tensorial del METRO orbitales, i H i , que tiene tenue 2 METRO .
Este producto tensorial no está simetrizado ni antisimetrizado. Es un producto tensorial simple. Ahora echemos un vistazo a TU H , que llamamos H 1 , el espacio de Hilbert de una sola partícula. está atravesado por METRO posibles estados ϕ 1 ( X ) , ϕ METRO ( X ) . podemos tener como mucho METRO electrones en este sistema, por lo que la dimensión del espacio de Hilbert completo debe ser finita. Contémoslo explícitamente. ¿Cuál es la dimensión del espacio de Hilbert de 2 partículas? Lo es H 1 H 1 ? No, porque estamos usando la primera notación cuantizada, y para obedecer el teorema de la estadística de espín, necesitamos antisimetrizar este espacio.
esto nos da H 2 = S v H 1 H 1 H 1 H 1 , dónde S v es el operador de antisimterización en un tensor. ¿Cuál es la dimensión de H 2 ? Es ( METRO 2 ) . Luego, de manera similar, tenue H 3 = ( METRO 3 ) , y así sucesivamente, hasta H METRO . Por último notamos que hay H 0 , el vacío, que NO tiene función de onda asociada (¡sin argumentos!), y tiene dimensión 1. Esto define el espacio de Fock como usted ha dicho, H = H 0 H 1 H METRO , que tiene tenue ( METRO 0 ) + ( METRO 1 ) + + ( METRO METRO ) = 2 METRO , ¡lo mismo de antes!
La diferencia clave, sin embargo, está en lo que H en realidad significa. No te confundas. Recuerda mi H es el espacio orbital de Hilbert, que tiene dim 2 . Su espacio de Hilbert de una sola partícula se ha atenuado METRO , que posiblemente puede ser infinito. Los operadores de creación y aniquilación dan la llamada segunda formulación de cuantización de QM, que evita (o más bien oculta) totalmente el problema de la antisimetrización en sus relaciones de conmutación. Ahí es donde te equivocas porque estás usando C a , C b como en la primera notación cuantizada.
No tengo la paciencia para leer todo el intercambio, pero me pregunto si la confusión depende del tipo de partición que estés haciendo . En ese caso voy a intentar comentar. los v = 1 El estado QH es puro bajo partición orbital, pero no bajo "partición de partículas" . Tal vez esto ayude: arxiv.org/abs/0905.4024 @nervxxx, su estado de 2 partículas puede ser puro bajo la partición orbital, pero está enredado bajo la partición de partículas. El enredo depende completamente de cómo elija particionar su sistema.
He elaborado ese comentario en mi respuesta.

Estoy publicando una versión modificada de mi comentario como respuesta, ya que más personas lo verán de esta manera.

Creo que la confusión depende de manera crucial del tipo de partición que esté haciendo . los v = 1 El estado QH es puro bajo partición orbital, pero no bajo "partición de partículas". Tal vez arXiv:0905.4204 ayude. IIRC, elaboran un ejemplo simple sobre este detalle, en la segunda sección.

@nervxxx, su estado de 2 partículas puede ser puro bajo la partición orbital, pero está enredado bajo la partición de partículas. Debido a la antisimetrización, parece un estado de campana singulete.

Entonces, la conclusión es que el enredo depende completamente de cómo elija particionar su sistema. La sutileza no es muy apreciada. Para una discusión detallada, consulte este artículo http://rspa.royalsocietypublishing.org/content/463/2085/2277.full

Si gracias. Eso es lo que concluí finalmente. Pero lo que me molesta (y la razón de todo el largo intercambio anterior) es que casi todos los demás no entienden el espacio de Fock (con el espacio de Hilbert de una sola partícula de dim METRO ) es isomorfo al producto tensorial de METRO espacios orbitales. Uno puede ver esto mirando la dimensión de los espacios de Hilbert - ambos son 2 METRO , y uno tiene un 1-1 entre los dos espacios, ¡entonces son isomorfos! Eso implica que uno puede descomponer el espacio de Fock en un producto tensorial, excepto que debido a que este isomorfismo solo funciona para los espacios completos,
no se puede descomponer la función de onda singlete en un producto tensorial como C a | 0 C b | 0 y representarlo como algunos F ( X 1 ) gramo ( X 2 ) .

Hay que tener cuidado con el formalismo bra-ket y su significado. A diferencia de | X 1 , no estoy seguro de que la notación | X 1 X 2 dónde X 1 y X 2 son coordenadas posicionales tiene algún sentido. En la literatura [1] la notación | a b designa el determinante de Slater o estado de Hartree-Fock, es decir:

| a b = C a C b | 0 = ϕ a ( X 1 ) ϕ b ( X 2 ) ϕ a ( X 2 ) ϕ b ( X 1 )

Mi sensación es que su confusión está relacionada con la mezcla del formalismo de los números de ocupación y la representación del espacio real.

[1] Szabo, Ostlund, "Química cuántica moderna: introducción a la teoría de la estructura electrónica avanzada"

Si alguien me explica el significado detrás de la notación |r1 r2>, estaré muy agradecido.
| x 1 x 2 es una abreviatura común para | x 1 | x2⟩ . _ _ En general, lo que está escrito dentro de un ket es solo una etiqueta: su significado depende en gran medida del contexto. Mezclar las dos representaciones no es raro, siempre que no haya confusión.
ok, estoy totalmente de acuerdo con eso. Pero consideremos un caso particular de la pregunta anterior, es decir, el caso cuando x 1 y x 2 son números cuánticos que etiquetan los estados propios del operador posicional ˆ x . ¿Cuál es el significado de descomponer \ket x 1\ket x 2 ? Además, tengo curiosidad por saber si hay alguna referencia que contenga dicha notación para el espacio de coordenadas. Sería interesante leer más al respecto.
Entendí x 1 y x 2 como etiquetas de la posición de los dos fermiones. Pero entonces, por supuesto, el ket \ket x 1 x 2 no está antisimetrizado y no corresponde a un estado permitido. Solo \ket x 1 x 2\ket x 2 x 1 es un estado válido.
¿Etiquetar significa números cuánticos? Creo que los símbolos en los vectores bra y ket en la notación de Dirac no pueden ser solo etiquetas. Deben ser números cuánticos adecuados. Por eso no le veo sentido claro a | x 1 x 2 > a menos que alguien me explique cuál es el estado del operador | x 1 x 2 > pertenecen?
En general, son etiquetas de vectores propios en el espacio de Hilbert. Si "números cuánticos propios" significa "etiquetas a la base propia de un operador dado", entonces no tienen que ser "números cuánticos propios". Si la base es gratuita, entonces no sé exactamente qué significa "adecuado".
Con Fermiones, la situación se complica por el hecho de que impones una simetría global. | x 1 x 2 | ↑↑ no es un estado válido, pero | x 1 x 2 ( | ↑↓ | ↓↑ es